Search found 14 matches


Thanks skang357 and Mike22629 for your response. I haven't taken the 1000 SC &CRs fully, only 1st 200s I took. I think I should seriously work on them to get the concepts accurate. And I started believing that I didn't have enough luck on that day, because my worst score from preptest itself is ...

by sasi78

Wed May 27, 2009 11:06 pm
Forum: GMAT Verbal & Essays
Topic: Disappointed with Verbal
Replies: 5
Views: 3440

Disappointed with Verbal

Hello, I need this forum help to diagnose what could have gone wrong with my verbal score. I wrote my GMAT exam on 13th May 09 and got score of 580 - much less than the minimum score from my prep tests. I consistently scored between 690 and 740 in Gmatprep tests, with verbal scores ranging from 35-4...

by sasi78

Sat May 23, 2009 12:55 am
Forum: GMAT Verbal & Essays
Topic: Disappointed with Verbal
Replies: 5
Views: 3440

1. not sufficient --> we cannot use given arc lenght because it might vary for different angles 2. not sufficient -- > not sufficient data given to compute cir. Both 1 & 2 sufficient -- > If r=s, then it becomes an equilateral triangle and we can make another similar equilateral triangle with XY...

by sasi78

Thu Apr 30, 2009 9:04 am
Forum: Data Sufficiency
Topic: Geometry problem - Curcumference of a circle
Replies: 7
Views: 2032

IMO the answer is E.

Reasons,
1. Parallelism "corrosion of metals" --> A,B&C are out
2. "includes" refer to singular only --> D is out.

by sasi78

Sun Jul 27, 2008 6:12 am
Forum: Sentence Correction
Topic: 476
Replies: 8
Views: 1995

as per quesiton, say the number d=0.abc

stmt 1: 10th digit of 10d is 7. So 10*d = a.bc and c=7. So C>5.
Stmt 2: 1000th digit of d/10 is 7. So d/10 = 0.0abc. This yields c=7. Agian C>5.

Hence, the answer is D. Either stmt is sufficient.

by sasi78

Sun Jun 08, 2008 6:36 am
Forum: Data Sufficiency
Topic: GMAT Prep DS num props with hundreths
Replies: 1
Views: 1296

Stem1 : n-3 = 2K can be written as n=2k+3, so there any multiple values for k & n and those may or may not be divisible by 7. so this is not sufficient. Stem2: 2K-4 is divisible by 7, so K = 9, 16 etc., but this doesn't tell anything about n. so this is also not sufficient. By using both stateme...

by sasi78

Sun Jun 08, 2008 5:38 am
Forum: Data Sufficiency
Topic: Is n divisible by 7?
Replies: 2
Views: 1769

This is my explanation

# of groups that have atleast on french teacher = total # of groups possible - # of groups that doesn't have any french teacher

= 10C3 - 6C3 = 120 - 20 = 100.

by sasi78

Sat Jun 07, 2008 11:24 pm
Forum: Data Sufficiency
Topic: gmat prep combinations
Replies: 3
Views: 1673

I feel, good understanding of the properties of different types of triangles suchs as isoceles & equilateral is must to answer this kind of quesitons.

by sasi78

Sat Jun 07, 2008 9:54 am
Forum: Data Sufficiency
Topic: Gprep Question - Isosceles
Replies: 4
Views: 1832

There was a formatting problem in my earlier reply. pl refer this. As per question, w+x<0. this can be writen as (w+x-y)<y then, (w-y)<-(x+y). Stem1: x+y<0. So (w-y)<x> 0. But we can't say (w-y) is <or> 0. Hence, this is not sufficient. Stem2: y<x<w. Add -y to all, 0<(x-y)<w>0. Hence, this is suffic...

by sasi78

Sat Jun 07, 2008 9:23 am
Forum: Data Sufficiency
Topic: inequalities
Replies: 7
Views: 2555

As per question,
w+x<0> w+x-y<y> w-y<-(x+y).

Stem1: x+y<0> w-y <x>0. But we can not say w-y is <or> 0. Hence, this is not sufficient.

Stem2: y<x<w> Add -y to all, 0<x-y<w> w-y >0. Hence, this is sufficient.

So, answer is B.

by sasi78

Sat Jun 07, 2008 9:16 am
Forum: Data Sufficiency
Topic: inequalities
Replies: 7
Views: 2555

Distance covered by cyclist in 5 mins = (20/60)*5 = 5/3 miles
Time taken for the hiker to cover 5/3 miles = (5/3)/(4/60) = 25 mins.

So, the answer is 25 mins.

Can someone confirm this pl.

by sasi78

Mon Jun 02, 2008 7:02 pm
Forum: Problem Solving
Topic: Tricky Speed/Distance question..
Replies: 6
Views: 4920

I think, no need to use POE for this kind of problems. Solving directly is the simple & less time consuming way.

by sasi78

Sun Jun 01, 2008 7:07 pm
Forum: Problem Solving
Topic: Is there any better approach to solve this Simple Question?
Replies: 5
Views: 1560

IMO, it is 7*4+1 = 29. what is the OA ??

by sasi78

Sun Jun 01, 2008 10:11 am
Forum: Problem Solving
Topic: help
Replies: 3
Views: 1458

Re: Pls,help me with this Question

the OA in Gmat800 is A : here is the explanation of the GMAT800 B and C are wrong because of the use of whose : whose can refer to canyon or layers. D is wordy : seen is better than been able to see. E is wrong because of the use of "making" : the subject og making is unclear : it can refe...

by sasi78

Thu May 01, 2008 9:12 am
Forum: Sentence Correction
Topic: Pls,help me with this Question
Replies: 1
Views: 3163